Menu Close

calculate-0-cos-chx-x-2-3-dx-




Question Number 83009 by mathmax by abdo last updated on 26/Feb/20
calculate ∫_0 ^∞   ((cos(chx))/(x^2  +3))dx
$${calculate}\:\int_{\mathrm{0}} ^{\infty} \:\:\frac{{cos}\left({chx}\right)}{{x}^{\mathrm{2}} \:+\mathrm{3}}{dx} \\ $$
Commented by mathmax by abdo last updated on 27/Feb/20
let A =∫_0 ^∞  ((cos(chx))/(x^2  +3))dx ⇒2A =∫_(−∞) ^(+∞)  ((cos(chx))/(x^2  +3))dx  =Re(∫_(−∞) ^(+∞)  (e^(ichx) /(x^2  +3))dx) let ϕ(z)=(e^(ich(z)) /(z^2  +3)) ⇒ϕ(z)=(e^(ich(z)) /((z−i(√3))(z+i(√3))))  residus theoem give  ∫_(−∞) ^(+∞)  ϕ(z)dz =2iπ Res(ϕ,i(√3)) =2iπ×(e^(ich(i(√3))) /(2i(√3)))  =(π/( (√3))) e^(i((e^(i(√3)) +e^(−i(√3)) )/2))  =(π/( (√3)))e^(i cos((√3)))  =(π/( (√3))){ cos(cos((√3)))+isin(cos((√3)))}  ⇒2A =(π/( (√3)))cos(cos((√3))) ⇒A =(π/(2(√3)))cos(cos((√3)))
$${let}\:{A}\:=\int_{\mathrm{0}} ^{\infty} \:\frac{{cos}\left({chx}\right)}{{x}^{\mathrm{2}} \:+\mathrm{3}}{dx}\:\Rightarrow\mathrm{2}{A}\:=\int_{−\infty} ^{+\infty} \:\frac{{cos}\left({chx}\right)}{{x}^{\mathrm{2}} \:+\mathrm{3}}{dx} \\ $$$$={Re}\left(\int_{−\infty} ^{+\infty} \:\frac{{e}^{{ichx}} }{{x}^{\mathrm{2}} \:+\mathrm{3}}{dx}\right)\:{let}\:\varphi\left({z}\right)=\frac{{e}^{{ich}\left({z}\right)} }{{z}^{\mathrm{2}} \:+\mathrm{3}}\:\Rightarrow\varphi\left({z}\right)=\frac{{e}^{{ich}\left({z}\right)} }{\left({z}−{i}\sqrt{\mathrm{3}}\right)\left({z}+{i}\sqrt{\mathrm{3}}\right)} \\ $$$${residus}\:{theoem}\:{give} \\ $$$$\int_{−\infty} ^{+\infty} \:\varphi\left({z}\right){dz}\:=\mathrm{2}{i}\pi\:{Res}\left(\varphi,{i}\sqrt{\mathrm{3}}\right)\:=\mathrm{2}{i}\pi×\frac{{e}^{{ich}\left({i}\sqrt{\mathrm{3}}\right)} }{\mathrm{2}{i}\sqrt{\mathrm{3}}} \\ $$$$=\frac{\pi}{\:\sqrt{\mathrm{3}}}\:{e}^{{i}\frac{{e}^{{i}\sqrt{\mathrm{3}}} +{e}^{−{i}\sqrt{\mathrm{3}}} }{\mathrm{2}}} \:=\frac{\pi}{\:\sqrt{\mathrm{3}}}{e}^{{i}\:{cos}\left(\sqrt{\mathrm{3}}\right)} \:=\frac{\pi}{\:\sqrt{\mathrm{3}}}\left\{\:{cos}\left({cos}\left(\sqrt{\mathrm{3}}\right)\right)+{isin}\left({cos}\left(\sqrt{\mathrm{3}}\right)\right)\right\} \\ $$$$\Rightarrow\mathrm{2}{A}\:=\frac{\pi}{\:\sqrt{\mathrm{3}}}{cos}\left({cos}\left(\sqrt{\mathrm{3}}\right)\right)\:\Rightarrow{A}\:=\frac{\pi}{\mathrm{2}\sqrt{\mathrm{3}}}{cos}\left({cos}\left(\sqrt{\mathrm{3}}\right)\right) \\ $$

Leave a Reply

Your email address will not be published. Required fields are marked *